0
$\begingroup$

The Riemann zeta function has the following representations for $$\text{Re}(s) > 1$$:

  • As a Dirichlet series: $$ \zeta(s) = \sum_{n=1}^\infty \frac{1}{n^s} $$

  • And as an Euler product: $$ \zeta(s) = \prod_{p \text{ prime}} \left(1 - \frac{1}{p^s} \right)^{-1} $$

These are equal only in the limit as $$N \to \infty$$.

However, I am not asking about approximations.

My question is this:

Can the finite sum $ \sum_{n=1}^N \frac{1}{n^s} $ be expressed as an exact finite product involving primes, possibly using the prime factorizations of integers up to N?

To clarify:

  • I am not asking for a truncated version of the Euler product as an approximation.
  • I'm asking whether there exists an exact expression of the finite sum $\sum_{n=1}^N \frac{1}{n^s}$ as a finite product over primes — possibly involving multiplicities, prime factorizations, or some clever rearrangement using prime powers.
  • In other words, is there a way to express the finite sum using a function similar in spirit to the Euler product, but in a way that is exact for finite $N$ and can be implemented programmatically to compute the sum precisely?
  • If such an expression exists, what is its form?

I understand that for the full zeta function, the infinite sum equals the infinite product. But I'm curious whether there is a meaningful finite product identity that holds exactly for the finite sum.

asked Oct 22 at 21:34
$\endgroup$
3
  • 1
    $\begingroup$ Hint: if $p$ is the greatest prime $\le N,ドル we need a 1ドル/(1-p^s)$ factor for the 1ドル/p^s$ term, but since 2ドルp>N$ there's a problem if $p\ge3$. $\endgroup$ Commented Oct 22 at 21:39
  • $\begingroup$ A reasonable question, but I'd think not, for slightly complicated (and not so interesting) reasons. $\endgroup$ Commented Oct 22 at 21:42
  • $\begingroup$ Paul Garrett, if I understand correctly, you’re saying that no answer to this question is known at present? $\endgroup$ Commented Oct 22 at 22:07

1 Answer 1

4
$\begingroup$

The answer is negative for $N \ge 3$ as follows.

Assume $$\sum_{n=1}^N \frac{1}{n^s}=\zeta_N(s)=\Pi_{p \le N}(\sum_{k \ge 0}a_k(p)p^{-ks})$$ where each term in the product converges somewhere. Then since there are finitely many terms in the product, they all converge absolutely for some $\Re s >A$ so we can switch sum and product and get that $a_0(p) \ne 0$ (free term is 1ドル=\Pi_P a_0(p)$) and $a_k(p)=0, p^k >N$ but $a_k(p) \ne 0$ if $p^k<N$ (just by looking at the $p^{-ks}$) terms on both sides.

In particular $a_1(p) \ne 0$ for all $p \le N$

But then picking $p\le N$ odd prime st 2ドルp>N$ which is possible when $N \ge 3$ we get that RHS contains a nonzero $(2p)^{-s}$ term and LHS doesn't and that is a contradiction!

answered Oct 22 at 22:48
$\endgroup$

You must log in to answer this question.

Start asking to get answers

Find the answer to your question by asking.

Ask question

Explore related questions

See similar questions with these tags.